Plz help I need help

Plz Help I Need Help

Answers

Answer 1

Answer:

a is 4 because 4 x 16=20 i hope this help

Answer 2
Answer :

a + 16 = 20

a = ?

a = 20 + 16

a = 36

a = 36 ✔️


Related Questions

I want to know what the letter a is , and how to do it.1/3 + a 5/4

Answers

STEP

1

:

           5

Simplify   —

           4

Equation at the end of step

1

:

  1          5

 (— +  a) -  —  = 0

  3          4

STEP

2

:

           1

Simplify   —

           3

Equation at the end of step

2

:

  1          5

 (— +  a) -  —  = 0

  3          4

STEP

3

:

Rewriting the whole as an Equivalent Fraction

3.1   Adding a whole to a fraction

Rewrite the whole as a fraction using  3  as the denominator :

        a     a • 3

   a =  —  =  —————

        1       3  

Equivalent fraction : The fraction thus generated looks different but has the same value as the whole

Common denominator : The equivalent fraction and the other fraction involved in the calculation share the same denominator

Adding fractions that have a common denominator :

3.2       Adding up the two equivalent fractions

Add the two equivalent fractions which now have a common denominator

Combine the numerators together, put the sum or difference over the common denominator then reduce to lowest terms if possible:

1 + a • 3     3a + 1

—————————  =  ——————

    3           3  

Equation at the end of step

3

:

 (3a + 1)    5

 ———————— -  —  = 0

    3        4

STEP

4

:

Calculating the Least Common Multiple :

4.1    Find the Least Common Multiple

     The left denominator is :       3

     The right denominator is :       4

       Number of times each prime factor

       appears in the factorization of:

Prime

Factor   Left

Denominator   Right

Denominator   L.C.M = Max

{Left,Right}

3 1 0 1

2 0 2 2

Product of all

Prime Factors  3 4 12

     Least Common Multiple:

     12

Calculating Multipliers :

4.2    Calculate multipliers for the two fractions

   Denote the Least Common Multiple by  L.C.M

   Denote the Left Multiplier by  Left_M

   Denote the Right Multiplier by  Right_M

   Denote the Left Deniminator by  L_Deno

   Denote the Right Multiplier by  R_Deno

  Left_M = L.C.M / L_Deno = 4

  Right_M = L.C.M / R_Deno = 3

Making Equivalent Fractions :

4.3      Rewrite the two fractions into equivalent fractions

Two fractions are called equivalent if they have the same numeric value.

For example :  1/2   and  2/4  are equivalent,  y/(y+1)2   and  (y2+y)/(y+1)3  are equivalent as well.

To calculate equivalent fraction , multiply the Numerator of each fraction, by its respective Multiplier.

  L. Mult. • L. Num.      (3a+1) • 4

  ——————————————————  =   ——————————

        L.C.M                 12    

  R. Mult. • R. Num.      5 • 3

  ——————————————————  =   —————

        L.C.M              12  

Adding fractions that have a common denominator :

4.4       Adding up the two equivalent fractions

(3a+1) • 4 - (5 • 3)     12a - 11

————————————————————  =  ————————

         12                 12  

Equation at the end of step

4

:

 12a - 11

 ————————  = 0

    12  

STEP

5

:

When a fraction equals zero :

5.1    When a fraction equals zero ...

Where a fraction equals zero, its numerator, the part which is above the fraction line, must equal zero.

Now,to get rid of the denominator, Tiger multiplys both sides of the equation by the denominator.

Here's how:

 12a-11

 —————— • 12 = 0 • 12

   12  

Now, on the left hand side, the  12  cancels out the denominator, while, on the right hand side, zero times anything is still zero.

The equation now takes the shape :

  12a-11  = 0

Solving a Single Variable Equation:

5.2      Solve  :    12a-11 = 0

Add  11  to both sides of the equation :

                     12a = 11

Divide both sides of the equation by 12:

                    a = 11/12 = 0.917

a = 11/12 = 0.917

                 

Answer:

11/12

Step-by-step explanation:

You want to subtract 5/4 minus 1/3 but you cant so you have to find the greatest denominator first which would be 12 because 3x4=12.

Since you multiplied 4x 3 to get 12 then you will multiply the 5 by 3 and the 1 by 4 to get this -> 5/4= 15/12     1/3= 4/12

Then subtract 15/12- 4/12= 11/12

a=11/12

Suppose Set A contains 19 elements and Set B contains 69 elements. If the total number elements
in either Set A or Set B is 75, how many elements do Sets A and B have in common?

Answers

B u A = 48

B = 45

The part of set B not included in A is B - A∩B  = 45 - 7 = 38

Since B u A = 48,  A must have 48-38 = 10 elements (3 exclusive of B, 7 shared with B).

------

Alternatively, the inclusion-exclusion principle may be used:   A  +  B  - (A ∩ B) = A u B  

  A + 45 - 7 = 48

  A  = 48 - 45 + 7  

   A = 10

NOTE that I'm using A and B  to indicate the number of elements in each set, respectively.

More formally, one would write |A|  and |B| to indicate the cardinality (number of elements) of the set.

3. Because it is so popular, a store owner increases the cost of a toy by $3.49. The new cost of the toy is $9.84.
(a) Write an equation that represents the situation. Use c to represent the original cost of the toy.
(b) Solve the equation using a related equation. Show your work.
(c) What does the solution of the equation represent? plz answer

Answers

a) 3.49+c=9.84 (b) 9.84-3.49=6.35 (c) he increased the original price of the toy by adding $6.35

Answer:

a) 3.49+c=9.84 (b) 9.84-3.49=6.35 (c) he increased the original price of the toy by adding $6.35. Just like jpforpresident146788 said.

What is the difference between (2.4x10^8) and (9.2x10^6)

Answers

Answer:

  2.308×10^8

Step-by-step explanation:

Your calculator can tell you what the difference is.

  2.4×10^8 -9.2×10^6 = (2.4 -0.092)×10^8 = 2.308×10^8

_____

Additional comment

If this is a problem where you're expected to round the answer to an appropriate number of significant figures, you will notice the largest contributor has a resolution of 0.1×10^8. That will be the resolution required in the answer: 2.3×10^8.

what is the perimeter of khgj?

Answers

Answer:

  58 in

Step-by-step explanation:

The fact that the rectangles are congruent means both have a length of 18 in and a width of 11 in. The perimeter is ...

  P = 2(L +W)

  P = 2(18 in + 11 in) = 2(29 in)

  P = 58 in

The perimeter of KHGJ is 58 inches.

0.9627281873+783,840,183=??!!
plzzz help​

Answers

ANSWER: 783,840,183.9627281873

EXPLANATION:
Just use your calculator lol

Proxima Centauri, our nearest star except for the sun, is about 4·1013 km from Earth. Find to one decimal place its distance in parsecs.

Answers

Step-by-step explanation:

The conversion for parsecs into meters is

1 parsec (pc) = [tex]3.09×10^{16}\:\text {m}[/tex]

So if Proxima Centauri is [tex]4×10^{13}\:\text{m}[/tex] away, then in terms of parsecs, this distance is

[tex]4×10^{13}\:\text{m}×\left(\dfrac{1\:\text{parsec}}{3.09×10^{16}\:\text{m}}\right)[/tex]

[tex]\;\;\;\;\;= 1.2×10^{-3}\:\text{parsec}[/tex]


A department store marked down all of its summer clothing 23%. The following week the remaining items were marked down again
15% off the sale price. When Jorge bought two tank tops on sale, he presented a coupon that gave him an additional 20% off. What
percent of the original price did Jorge save?

Answers

Answer:

58%

Step-by-step explanation:

You get brainist plz

Answers

Answer:

54

Step-by-step explanation:

54

What is 162 multiplied by 0.967

Answers

The answer would be 156.654

Answer:

Remove the decimals and multiply normally.

162 x 967

156654

Put back in the decimals. In this case, move the decimal point back three spaces from the end.

156.654

:)

WILL GIVE BRAINLEST! HELP ME ASAP

These tables of ordered pairs represent some points on the graphs of lines s and t.
Which system of equations is represented by lines s and t?

5x−2y=−157x+4y=20

3x+5y=−185x−9y=25

x+4y=−89x−y=21

2x−y=−142x+y=6

Answers

Answer:

Step-by-step explanation:

Line s

slope m = Δy / Δx =  (22 - 4) / (4 - (-5)) = 18/9 = 2

y = mx + b

22 = 2(4) + b

b = 14

y = 2x + 14

which can be rearranged to

2x - y = -14

Line t

slope = ( -4 - 14) / (5 - (-4)) = -18/9 = -2

-4 = (-2)(5) + b

b = 6

y = -2x + 6

2x + y = 6

2x − y = −14             2x + y = 6

Help I’ll mark you brainlist

Answers

Answer:

24.0

Step-by-step explanation:

do 34sin(45) to solve for x

2-2 this is a freebie for points guys

Answers

Answer:

2-2=0

Step-by-step explanation:

You have 2 cookies and you give it to 2 friends.

Answer:

0

Step-by-step explanation:

According to the Laws of Physics, yeah I don't wanna try explaining this

I'll give brainiest whoever answers first

Answers

aight slide it here big boy

Find the distance from point $A\left(15,-21\right)$ to the line $5x+2y\ =\ 4$ . Round your answer to the nearest tenth.

Answers

The distance from point (15,-21) to the line 5x + 2y = 4 is 27.5 units

Given the coordinate (15, -21) and the line 5x + 2y = 4

In order to get the point on the line 5x + 2y =4, we can a point on the line

Let x = 0

5(0) + 2y = 4

2y = 4

y = 2

The point (0, 2) is on the line.

Find the distance between the point (15, -21) and (0, 2) using the distance formula

[tex]D=\sqrt{(2+21)^2+(0-15)^2} \\D=\sqrt{23^2+15^2}\\D=\sqrt{754}\\D= 27.46units[/tex]

Hence the distance from point (15,-21) to the line 5x + 2y = 4 is 27.5 units

Learn more here: https://brainly.com/question/22624745

2. Solve the equation and check.
8x-8= 7 + 3x
Select the correct choice below and, if necessary, fill in the answer box to complete your choice.

Answers

the answer is x=3 i believe

Which fraction converts to a repeating decimal number?

Answers

Answer:

1/3

Step-by-step explanation:

solve pls brainliest

Answers

Answer:

Mixed Number = 2 9/10

Improper Fraction = 29/10

Step-by-step explanation:

Mixed Number = 29/10 = 2 9/10

Improper Fraction = 29/10

Evaluate 11/6 - 2/8=

Answers

Answer:

19/12

Step-by-step explanation:

The sum of 13 and 3 times a number is 31. What is the number?

Answers

Answer:

6

Step-by-step explanation:

yeah-ya ............... right?

Answer: 6

Step-by-step explanation:

13 + 3 × n = 31

n represents unknown number

13 + 3n = 31

-13           -13

3n = 18

/3      /3

n=6

How many tiles would be used to make figure five

Answers

attach a pic please
that would help



How have vaccines impacted health?


A: Vaccines have only made a little progress at preventing most diseases.
B: Vaccines are effective on some people but not others.
C: Large numbers of people have had allergic reactions to vaccines
D: Some diseases have almost completely disappeared



Answers

Answer:

D some disease have almost completely disappeared

B. Or D. Would be your answer. The reason why I would say B. Is because people have different immune systems which makes your body reject the vaccine contents. But on the other hand for D. Is that it they have made a lot of pretty bad sicknesses almost disappear such as chicken pox.

Last year at a certain high school, there were 100 boys on the honor roll and 60 girls on the honor roll. This year, the number of boys on the honor roll decreased by 11% and the number of girls on the honor roll decreased by 10%. By what percentage did the total number of students on the honor roll increase? Round your answer to the nearest tenth (if necessary).​

Answers

Answer:

-11 boys -6 girls

Step-by-step explanation:

(1/2)power 5=?? please help

Answers

[tex] \frac{1}{2} \: to \: the \: power \: of \: 5 \: is \: \frac{1}{32} or \: 0.03125 [/tex]

Answer:

[tex] \ {\boxed{ \tt ( \frac{1}{2}) ^{5} = \frac{1}{32} }}[/tex]

Step-by-step explanation:

[tex] \\ \tt( \frac{1}{2} )^{5} [/tex]

[tex] \\ \tt= \frac{ {1}^{5} }{ {2}^{5} } [/tex]

[tex] \\ \tt = \frac{1 \times 1 \times 1 \times 1 \times 1}{2 \times 2 \times 2 \times 2 \times 2} [/tex]

[tex] \\ \tt = \frac{1}{32} [/tex]

Answer this please not links-

Answers

Answer:

I believe it's 9 hope this helps

Help pls


A hot-air balloon is 1550 feet off the ground and its altitude is slowly changing at a constant rate of −7 1/2 feet per second.

How many seconds, s, will it take for the balloon to drop to below 465 feet?

Drag and drop a symbol and value to correctly complete the solution to this inequality.

Answers

Answer:

s> 144 2/3

Step-by-step explanation:

I took the test in k12 and i got it right so yea !

Answer: the other person was right

Step-by-step explanation:

Historically, 48% of people have rented their residence. A random sample of 500 people is taken. 260 of them rent (as oppose to own) their residence. A researcher is studying the claim that the percentage of people renting has increased. Compute the value of the test statistic for this hypothesis test. Round your answer to 2 decimal places.

Answers

Answer:

1.79

Step-by-step explanation:

^p = x/n

    = 260/500

    = 0.52

z = p1-p2 / √^p(1-p)/n

  = 0.52 - 0.48 / √0.48x(1-0.48)/500

  = 1.79

dur rn help pls ))))))))))

Answers

Answer:

A is correct good luck bud

Step-by-step explanation:

What is the sum of all positive integers n ≤ 20 that satisfy the condition that if n is odd, then n is a multiple of 5? I will give brainliest for the correct answer AND explanation (It is not 20)

Answers

The sum of all positive integers that satisfy the condition as described is; 100

The number set of n is therefore;

n = {1, 2,3,4,5,6,7,8,9,10,11,12,13,14,15,16,17,18,19,20}.

According to the question;

If n is odd, then n is a multiple of 5.

The data set therefore includes all odd numbers; This is so because the question says; if n is odd; then n is a multiple of 5.

The sum of all positive integers that satisfy the condition is therefore;

1 + 3 +5 +7 + 9 +11 +13 +15 +17 +19 = 100.

Read more:

https://brainly.com/question/18837188

PLEASE HELP!!
Guys please help i need this

Answers

Answer:

Because they are both base angles of isosceles triangles

Other Questions
WILL MARK BRAINLIEST IF WORK IS SHOWN Mr. Jackson's dog Gromit jumped out of a second story window (through the screen) and fell to the ground below. The window is 15 feet high, the acceleration of gravity is 9.8m/s2, and there are 3.28 feet in each meter. What was the final velocity of this crazed dog? how do ectotherms primarily control their body temperature? Determine whether a quadratic model exists for the set of values below. If so, write the model. f(0) = -4, f(3) = -19, f(-1) = -11Select the correct choice below:A) A quadratic model does exist. It is f(x)= ?B) A quadratic model does not exist. (Help ASAP please!!) Which statement best explains how the third-person omniscient point of view affects "To Build a Fire"?It allows the reader to see the contrast between the dog's sensible thoughts and the man's illogical thoughts.It gives the reader access to only the man's thoughts and feelings, which makes the reader realize how overconfident he is.It allows the reader to experience the story through the man's view, which allows readers to feel compassion for this character.It uses the pronoun "you" to place the reader directly in the story, which adds to the theme of loneliness in nature. Why is there anargument about thisamendment today? A Norman window is a window with a semi-circle on top of regular rectangular window. What should be the dimensions of the rectangular part of the norman window to allow in as much light as possible, if there are only 12 ft of the frame material. A scientist has two solutions, which she has labeled Solution A and Solution B. Each contains salt. She knows that Solution A is 45% salt and Solution B is95% salt. She wants to obtain 60 ounces of a mixture that is 85% salt. How many ounces of each solution should she use? CAN YALL GO FOLLOW MY INS PLSS yeabdagod What is the slope of the line y=5-4x ?Show your work Please help, I will give brainly if correct!***Pls provide source if possible, thank you!***How did Latin American relations with the United States change after World War II? What type of population distribution pattern is exhibited by a flock of birds that nest close together? The substances on the left side of a chemical equation are called the:Select one:a.Ingredientsb.Reactantsc.Components Suppose you roll a pair of six-sided dice and add their totals, What is the probability that the sum of the numbers on your dice is a number other than 11 Driveway pavers are 6 inches by 10 inches. If your driveway is 21840 square inches, how many pavers do you need for your driveway? Correct the verb between brackets:Fahad Has (drive ) since he was 10 Tell whether x and y show direct variation x= (y-2)/(9)PLEASE HELP I"LL MARK BRAINLIESTEXPLAIN PLS Three teams (Team A, Team B, and Team C) are participating in a trivia contest. Let scoreA represent the number of correct questions for Team A, scoreB represent the number of correct questions for Team B, and scoreC represent the number of correct questions for Team C. Assuming no two teams get the same number of correct questions, what code segments correctly displays the team with the highest number of correct questions? A fish swims 4 feet directly above a scuba diver. The fish suddenly dives to double its depth under the surface of the ocean. The fish dives to a depth of 34 feet.What is the position of the scuba diver relative to sea level?Choose from the drop-down menu to correctly complete the statement.The scuba diver is Choose... feet Choose... sea level. What is the answer for 14.8 x -2.5 The populations of Peru and Bolivia are the only nations in South America that have a majority of native citizens; while the remaining nations' citizens are a mix of native and European nationalities.true or false